Does there exist a group that is both a free product and a direct product of nontrivial groups?












4












$begingroup$


Do there exist such nontrivial groups $A$, $B$, $C$ and $D$, such that $A times B cong C ast D$?



I failed to construct any examples, so I decided to try to prove they do not exist by contradiction.



If such groups exist, then $C$ and $D$ are disjoint subgroups of $A times B$. Suppose $w in F[x, y] {e}$, where $F[x, y]$ is the free group with generators $x$ and $y$. Suppose $(a_c, b_c) in C$, $(a_d, b_d) in D$ and $h: F[x, y] rightarrow A times B$ is a homomorphism, that maps $x$ to $(a_c, b_c)$ and $y$ to $(a_d, b_d)$. Then, by definition of free product $h(w) neq e$. Thus, either $pi_A(h(w)) neq e$ or $pi_B(h(w)) neq e$, where $pi_A$ and $pi_B$ are projections on $A times B$ onto $A$ and $B$ respectively. Thus every group word is not an identity either for $A$ or for $B$ and that results in ${A, B}$ generating the variety of all groups. And here I am stuck, failing to determine anything else.



Or do the examples actually exist?










share|cite|improve this question











$endgroup$








  • 1




    $begingroup$
    Related: mathoverflow.net/questions/253052/… and math.stackexchange.com/questions/78227/…
    $endgroup$
    – Michael Burr
    6 hours ago


















4












$begingroup$


Do there exist such nontrivial groups $A$, $B$, $C$ and $D$, such that $A times B cong C ast D$?



I failed to construct any examples, so I decided to try to prove they do not exist by contradiction.



If such groups exist, then $C$ and $D$ are disjoint subgroups of $A times B$. Suppose $w in F[x, y] {e}$, where $F[x, y]$ is the free group with generators $x$ and $y$. Suppose $(a_c, b_c) in C$, $(a_d, b_d) in D$ and $h: F[x, y] rightarrow A times B$ is a homomorphism, that maps $x$ to $(a_c, b_c)$ and $y$ to $(a_d, b_d)$. Then, by definition of free product $h(w) neq e$. Thus, either $pi_A(h(w)) neq e$ or $pi_B(h(w)) neq e$, where $pi_A$ and $pi_B$ are projections on $A times B$ onto $A$ and $B$ respectively. Thus every group word is not an identity either for $A$ or for $B$ and that results in ${A, B}$ generating the variety of all groups. And here I am stuck, failing to determine anything else.



Or do the examples actually exist?










share|cite|improve this question











$endgroup$








  • 1




    $begingroup$
    Related: mathoverflow.net/questions/253052/… and math.stackexchange.com/questions/78227/…
    $endgroup$
    – Michael Burr
    6 hours ago
















4












4








4


2



$begingroup$


Do there exist such nontrivial groups $A$, $B$, $C$ and $D$, such that $A times B cong C ast D$?



I failed to construct any examples, so I decided to try to prove they do not exist by contradiction.



If such groups exist, then $C$ and $D$ are disjoint subgroups of $A times B$. Suppose $w in F[x, y] {e}$, where $F[x, y]$ is the free group with generators $x$ and $y$. Suppose $(a_c, b_c) in C$, $(a_d, b_d) in D$ and $h: F[x, y] rightarrow A times B$ is a homomorphism, that maps $x$ to $(a_c, b_c)$ and $y$ to $(a_d, b_d)$. Then, by definition of free product $h(w) neq e$. Thus, either $pi_A(h(w)) neq e$ or $pi_B(h(w)) neq e$, where $pi_A$ and $pi_B$ are projections on $A times B$ onto $A$ and $B$ respectively. Thus every group word is not an identity either for $A$ or for $B$ and that results in ${A, B}$ generating the variety of all groups. And here I am stuck, failing to determine anything else.



Or do the examples actually exist?










share|cite|improve this question











$endgroup$




Do there exist such nontrivial groups $A$, $B$, $C$ and $D$, such that $A times B cong C ast D$?



I failed to construct any examples, so I decided to try to prove they do not exist by contradiction.



If such groups exist, then $C$ and $D$ are disjoint subgroups of $A times B$. Suppose $w in F[x, y] {e}$, where $F[x, y]$ is the free group with generators $x$ and $y$. Suppose $(a_c, b_c) in C$, $(a_d, b_d) in D$ and $h: F[x, y] rightarrow A times B$ is a homomorphism, that maps $x$ to $(a_c, b_c)$ and $y$ to $(a_d, b_d)$. Then, by definition of free product $h(w) neq e$. Thus, either $pi_A(h(w)) neq e$ or $pi_B(h(w)) neq e$, where $pi_A$ and $pi_B$ are projections on $A times B$ onto $A$ and $B$ respectively. Thus every group word is not an identity either for $A$ or for $B$ and that results in ${A, B}$ generating the variety of all groups. And here I am stuck, failing to determine anything else.



Or do the examples actually exist?







abstract-algebra group-theory direct-product combinatorial-group-theory free-product






share|cite|improve this question















share|cite|improve this question













share|cite|improve this question




share|cite|improve this question








edited 7 hours ago









Shaun

9,183113683




9,183113683










asked 7 hours ago









Yanior WegYanior Weg

1,94311140




1,94311140








  • 1




    $begingroup$
    Related: mathoverflow.net/questions/253052/… and math.stackexchange.com/questions/78227/…
    $endgroup$
    – Michael Burr
    6 hours ago
















  • 1




    $begingroup$
    Related: mathoverflow.net/questions/253052/… and math.stackexchange.com/questions/78227/…
    $endgroup$
    – Michael Burr
    6 hours ago










1




1




$begingroup$
Related: mathoverflow.net/questions/253052/… and math.stackexchange.com/questions/78227/…
$endgroup$
– Michael Burr
6 hours ago






$begingroup$
Related: mathoverflow.net/questions/253052/… and math.stackexchange.com/questions/78227/…
$endgroup$
– Michael Burr
6 hours ago












3 Answers
3






active

oldest

votes


















3












$begingroup$

Let me just replicate YCor's comment in the link that was given in the comments, with more details :




in a free product $Cast D$ with $C,D$ nontrivial, the intersection of any two nontrivial normal subgroups is nontrivial.




Indeed, let $H,K$ be two nontrivial normal subgroups of $C*D$. I'll assume for simplicity that $|C|,|D|$ are large enough so that for each $x,y$ there is a nontrivial $z$ with $z^{-1}neq x, zneq y$. For instance $|C|, |D|geq 4$ is good enough (take $x,y$, then there are at most two nontrivial $z$ such that $z=x$ or $z^{-1}=y$ : $x$ and $y^{-1}$; so if $|G|geq 4$, any nontrivial element different from $x$ and $y^{-1}$ works)



Let me say that $c_1d_1....c_nd_n$ is the reduced form of an element of $C*D$ if $c_iin C, d_jin D$, and the only $c_i,d_j$ allowed to be $1$ are $c_1$ and $d_n$. Clearly, if $x=c_1d_1....c_nd_n$ is the reduced form of $x$, and $ngeq 2$, then $xneq 1$ in $C*D$ ("clearly" here is to be understood as : it's a classical property of free products), moreover, if $n=1$ this is $1$ if and only if $c_1=d_1=1$.



Now let $x= c_1d_1...c_rd_r in H, y=c'_1d'_1...c'_sd'_s in K$ be nontrivial elements, with obvious notations, written in reduced form. The point will be that $[x,y]in Hcap K$ (this is obvious), and that, up to changing $x,y$ a bit, this can't be the trivial element.



Now up to conjugation by some element of $C$, one may assume $d_r = 1$ and $c_1neq 1$ (this can be done by the hypothesis on $|C|$ - and $c_r neq 1$, but that follows from the reduced form); and up to conjugation by some element of $D$, $c'_1 = 1$ , $d_s'neq 1$ (using the cardinality hypothesis on $D$ - and $d_1' neq 1$, but again this follows from the reduced form).



So with these hypotheses $[x,y] = c_1d_1...c_rd_1'....c_s'd_s'c_r^{-1}....d_1^{-1}c_1^{-1}d_s'^{-1}c_s'^{-1}...d_1'^{-1}$ which is written in reduced form, and is thus $neq 1$. Therefore $[x,y]in Hcap Ksetminus{1}$.



Apply this to your supposed $Atimes {1}, {1}times B$ to get a contradiction.



I don't know if there's an easier argument, or a not-too-complicated argument that encapsulates the low cardinality cases, but I guess for these you have to go "by hand" somehow; or perhaps you can adapt this argument to these cases by working a bit more. In any case I didn't want to bother with these cases, and this argument works in most cases and is pretty painless so in any case it's interesting to share






share|cite|improve this answer











$endgroup$





















    3












    $begingroup$

    Let $f:Atimes Brightarrow C*D$ be an isomorphism where $A,B,C,D$ are non trivial groups. Let $a_0in A$ not trivial, for every $bin B$, $f(a_0)$ commutes with $f(b)$.



    Now we look the proof of the The Corollary 4.1.6 p. 187 of Magnus Karrass and Solitar.



    The first part of this proof asserts that if $f(a_0)$ is contained in the conjugated of a free factor, that is $f(a_0)$ is in $gCg^{-1}$ or is in $gDg^{-1}$ then so is $f(b)$ for every $bin B$. Without restricting the generality, we suppose that $f(a_0)$ and so $f(B)$ are contained in $gCg^{-1}$. Let $b_0$ a non trivial element of $B$ since $B$ is contained in the free conjugated factor $gCg^{-1}$ the same argument shows that $f(A)$ is contained in $gCg^{-1}$ this implies that $C*D$ is contained in a free conjugated factor. Contradiction.



    The second part of the proof shows that if $f(a_0)$ is not contained in a free conjugated factor, then there exists $u_c$ such $f(a_0)$ and $f(b)$ are a power of $u_c$.



    You can express $f(a_0)$ uniquely as a reduced sequence (Theorem 4.1.) this implies that there exists a unique element $u$ with minimal length such that $f(a_0)$ is a power of $u$ and if $f(a_0)$ is a power of $v$, $v$ is a power of $u$. This impllies that every element of $f(B)$ are power of $u$ and $f(B)$ and $B$ are cyclic. A similar argument shows that $f(A)$ and $A$ are cyclic, we deduce that $Atimes B$ is commutative. Contradicition since $C*D$ is not commutative.



    Reference.



    Combinatorial Group theory.



    Magnus, Karrass and Solitar.






    share|cite|improve this answer











    $endgroup$





















      0












      $begingroup$

      To be a free product $Cast D$ implies the existence of an action on oriented tree such that $C$ is the stabilizer of some vertex $v_0$ and such that edge stabilizers are trivial (so that nontrivial element fix at most one vertex).



      Let $(a,b)$ be a nontrivial element of $C$; we can suppose that $aneq 1$ up to switch $A$ and $B$. Then in the Bass-Serre tree, $(a,b)$ fixes a unique vertex $v_0$, and hence this vertex is also fixed by the centralizer of $(a,b)$, and hence $(a,1)$ fixes $v_0$. Applying this to $(a,1)$ shows that $B$ fixes the vertex $v_0$. Choose $1neq b'in B$. It fixes the unique vertex $v_0$ and hence its centralizer fixes $v_0$, so $A$ fixes $v_0$. Finally $G=Atimes B$ fixes $v_0$, so $D=1$.






      share|cite|improve this answer









      $endgroup$













        Your Answer





        StackExchange.ifUsing("editor", function () {
        return StackExchange.using("mathjaxEditing", function () {
        StackExchange.MarkdownEditor.creationCallbacks.add(function (editor, postfix) {
        StackExchange.mathjaxEditing.prepareWmdForMathJax(editor, postfix, [["$", "$"], ["\\(","\\)"]]);
        });
        });
        }, "mathjax-editing");

        StackExchange.ready(function() {
        var channelOptions = {
        tags: "".split(" "),
        id: "69"
        };
        initTagRenderer("".split(" "), "".split(" "), channelOptions);

        StackExchange.using("externalEditor", function() {
        // Have to fire editor after snippets, if snippets enabled
        if (StackExchange.settings.snippets.snippetsEnabled) {
        StackExchange.using("snippets", function() {
        createEditor();
        });
        }
        else {
        createEditor();
        }
        });

        function createEditor() {
        StackExchange.prepareEditor({
        heartbeatType: 'answer',
        autoActivateHeartbeat: false,
        convertImagesToLinks: true,
        noModals: true,
        showLowRepImageUploadWarning: true,
        reputationToPostImages: 10,
        bindNavPrevention: true,
        postfix: "",
        imageUploader: {
        brandingHtml: "Powered by u003ca class="icon-imgur-white" href="https://imgur.com/"u003eu003c/au003e",
        contentPolicyHtml: "User contributions licensed under u003ca href="https://creativecommons.org/licenses/by-sa/3.0/"u003ecc by-sa 3.0 with attribution requiredu003c/au003e u003ca href="https://stackoverflow.com/legal/content-policy"u003e(content policy)u003c/au003e",
        allowUrls: true
        },
        noCode: true, onDemand: true,
        discardSelector: ".discard-answer"
        ,immediatelyShowMarkdownHelp:true
        });


        }
        });














        draft saved

        draft discarded


















        StackExchange.ready(
        function () {
        StackExchange.openid.initPostLogin('.new-post-login', 'https%3a%2f%2fmath.stackexchange.com%2fquestions%2f3110589%2fdoes-there-exist-a-group-that-is-both-a-free-product-and-a-direct-product-of-non%23new-answer', 'question_page');
        }
        );

        Post as a guest















        Required, but never shown

























        3 Answers
        3






        active

        oldest

        votes








        3 Answers
        3






        active

        oldest

        votes









        active

        oldest

        votes






        active

        oldest

        votes









        3












        $begingroup$

        Let me just replicate YCor's comment in the link that was given in the comments, with more details :




        in a free product $Cast D$ with $C,D$ nontrivial, the intersection of any two nontrivial normal subgroups is nontrivial.




        Indeed, let $H,K$ be two nontrivial normal subgroups of $C*D$. I'll assume for simplicity that $|C|,|D|$ are large enough so that for each $x,y$ there is a nontrivial $z$ with $z^{-1}neq x, zneq y$. For instance $|C|, |D|geq 4$ is good enough (take $x,y$, then there are at most two nontrivial $z$ such that $z=x$ or $z^{-1}=y$ : $x$ and $y^{-1}$; so if $|G|geq 4$, any nontrivial element different from $x$ and $y^{-1}$ works)



        Let me say that $c_1d_1....c_nd_n$ is the reduced form of an element of $C*D$ if $c_iin C, d_jin D$, and the only $c_i,d_j$ allowed to be $1$ are $c_1$ and $d_n$. Clearly, if $x=c_1d_1....c_nd_n$ is the reduced form of $x$, and $ngeq 2$, then $xneq 1$ in $C*D$ ("clearly" here is to be understood as : it's a classical property of free products), moreover, if $n=1$ this is $1$ if and only if $c_1=d_1=1$.



        Now let $x= c_1d_1...c_rd_r in H, y=c'_1d'_1...c'_sd'_s in K$ be nontrivial elements, with obvious notations, written in reduced form. The point will be that $[x,y]in Hcap K$ (this is obvious), and that, up to changing $x,y$ a bit, this can't be the trivial element.



        Now up to conjugation by some element of $C$, one may assume $d_r = 1$ and $c_1neq 1$ (this can be done by the hypothesis on $|C|$ - and $c_r neq 1$, but that follows from the reduced form); and up to conjugation by some element of $D$, $c'_1 = 1$ , $d_s'neq 1$ (using the cardinality hypothesis on $D$ - and $d_1' neq 1$, but again this follows from the reduced form).



        So with these hypotheses $[x,y] = c_1d_1...c_rd_1'....c_s'd_s'c_r^{-1}....d_1^{-1}c_1^{-1}d_s'^{-1}c_s'^{-1}...d_1'^{-1}$ which is written in reduced form, and is thus $neq 1$. Therefore $[x,y]in Hcap Ksetminus{1}$.



        Apply this to your supposed $Atimes {1}, {1}times B$ to get a contradiction.



        I don't know if there's an easier argument, or a not-too-complicated argument that encapsulates the low cardinality cases, but I guess for these you have to go "by hand" somehow; or perhaps you can adapt this argument to these cases by working a bit more. In any case I didn't want to bother with these cases, and this argument works in most cases and is pretty painless so in any case it's interesting to share






        share|cite|improve this answer











        $endgroup$


















          3












          $begingroup$

          Let me just replicate YCor's comment in the link that was given in the comments, with more details :




          in a free product $Cast D$ with $C,D$ nontrivial, the intersection of any two nontrivial normal subgroups is nontrivial.




          Indeed, let $H,K$ be two nontrivial normal subgroups of $C*D$. I'll assume for simplicity that $|C|,|D|$ are large enough so that for each $x,y$ there is a nontrivial $z$ with $z^{-1}neq x, zneq y$. For instance $|C|, |D|geq 4$ is good enough (take $x,y$, then there are at most two nontrivial $z$ such that $z=x$ or $z^{-1}=y$ : $x$ and $y^{-1}$; so if $|G|geq 4$, any nontrivial element different from $x$ and $y^{-1}$ works)



          Let me say that $c_1d_1....c_nd_n$ is the reduced form of an element of $C*D$ if $c_iin C, d_jin D$, and the only $c_i,d_j$ allowed to be $1$ are $c_1$ and $d_n$. Clearly, if $x=c_1d_1....c_nd_n$ is the reduced form of $x$, and $ngeq 2$, then $xneq 1$ in $C*D$ ("clearly" here is to be understood as : it's a classical property of free products), moreover, if $n=1$ this is $1$ if and only if $c_1=d_1=1$.



          Now let $x= c_1d_1...c_rd_r in H, y=c'_1d'_1...c'_sd'_s in K$ be nontrivial elements, with obvious notations, written in reduced form. The point will be that $[x,y]in Hcap K$ (this is obvious), and that, up to changing $x,y$ a bit, this can't be the trivial element.



          Now up to conjugation by some element of $C$, one may assume $d_r = 1$ and $c_1neq 1$ (this can be done by the hypothesis on $|C|$ - and $c_r neq 1$, but that follows from the reduced form); and up to conjugation by some element of $D$, $c'_1 = 1$ , $d_s'neq 1$ (using the cardinality hypothesis on $D$ - and $d_1' neq 1$, but again this follows from the reduced form).



          So with these hypotheses $[x,y] = c_1d_1...c_rd_1'....c_s'd_s'c_r^{-1}....d_1^{-1}c_1^{-1}d_s'^{-1}c_s'^{-1}...d_1'^{-1}$ which is written in reduced form, and is thus $neq 1$. Therefore $[x,y]in Hcap Ksetminus{1}$.



          Apply this to your supposed $Atimes {1}, {1}times B$ to get a contradiction.



          I don't know if there's an easier argument, or a not-too-complicated argument that encapsulates the low cardinality cases, but I guess for these you have to go "by hand" somehow; or perhaps you can adapt this argument to these cases by working a bit more. In any case I didn't want to bother with these cases, and this argument works in most cases and is pretty painless so in any case it's interesting to share






          share|cite|improve this answer











          $endgroup$
















            3












            3








            3





            $begingroup$

            Let me just replicate YCor's comment in the link that was given in the comments, with more details :




            in a free product $Cast D$ with $C,D$ nontrivial, the intersection of any two nontrivial normal subgroups is nontrivial.




            Indeed, let $H,K$ be two nontrivial normal subgroups of $C*D$. I'll assume for simplicity that $|C|,|D|$ are large enough so that for each $x,y$ there is a nontrivial $z$ with $z^{-1}neq x, zneq y$. For instance $|C|, |D|geq 4$ is good enough (take $x,y$, then there are at most two nontrivial $z$ such that $z=x$ or $z^{-1}=y$ : $x$ and $y^{-1}$; so if $|G|geq 4$, any nontrivial element different from $x$ and $y^{-1}$ works)



            Let me say that $c_1d_1....c_nd_n$ is the reduced form of an element of $C*D$ if $c_iin C, d_jin D$, and the only $c_i,d_j$ allowed to be $1$ are $c_1$ and $d_n$. Clearly, if $x=c_1d_1....c_nd_n$ is the reduced form of $x$, and $ngeq 2$, then $xneq 1$ in $C*D$ ("clearly" here is to be understood as : it's a classical property of free products), moreover, if $n=1$ this is $1$ if and only if $c_1=d_1=1$.



            Now let $x= c_1d_1...c_rd_r in H, y=c'_1d'_1...c'_sd'_s in K$ be nontrivial elements, with obvious notations, written in reduced form. The point will be that $[x,y]in Hcap K$ (this is obvious), and that, up to changing $x,y$ a bit, this can't be the trivial element.



            Now up to conjugation by some element of $C$, one may assume $d_r = 1$ and $c_1neq 1$ (this can be done by the hypothesis on $|C|$ - and $c_r neq 1$, but that follows from the reduced form); and up to conjugation by some element of $D$, $c'_1 = 1$ , $d_s'neq 1$ (using the cardinality hypothesis on $D$ - and $d_1' neq 1$, but again this follows from the reduced form).



            So with these hypotheses $[x,y] = c_1d_1...c_rd_1'....c_s'd_s'c_r^{-1}....d_1^{-1}c_1^{-1}d_s'^{-1}c_s'^{-1}...d_1'^{-1}$ which is written in reduced form, and is thus $neq 1$. Therefore $[x,y]in Hcap Ksetminus{1}$.



            Apply this to your supposed $Atimes {1}, {1}times B$ to get a contradiction.



            I don't know if there's an easier argument, or a not-too-complicated argument that encapsulates the low cardinality cases, but I guess for these you have to go "by hand" somehow; or perhaps you can adapt this argument to these cases by working a bit more. In any case I didn't want to bother with these cases, and this argument works in most cases and is pretty painless so in any case it's interesting to share






            share|cite|improve this answer











            $endgroup$



            Let me just replicate YCor's comment in the link that was given in the comments, with more details :




            in a free product $Cast D$ with $C,D$ nontrivial, the intersection of any two nontrivial normal subgroups is nontrivial.




            Indeed, let $H,K$ be two nontrivial normal subgroups of $C*D$. I'll assume for simplicity that $|C|,|D|$ are large enough so that for each $x,y$ there is a nontrivial $z$ with $z^{-1}neq x, zneq y$. For instance $|C|, |D|geq 4$ is good enough (take $x,y$, then there are at most two nontrivial $z$ such that $z=x$ or $z^{-1}=y$ : $x$ and $y^{-1}$; so if $|G|geq 4$, any nontrivial element different from $x$ and $y^{-1}$ works)



            Let me say that $c_1d_1....c_nd_n$ is the reduced form of an element of $C*D$ if $c_iin C, d_jin D$, and the only $c_i,d_j$ allowed to be $1$ are $c_1$ and $d_n$. Clearly, if $x=c_1d_1....c_nd_n$ is the reduced form of $x$, and $ngeq 2$, then $xneq 1$ in $C*D$ ("clearly" here is to be understood as : it's a classical property of free products), moreover, if $n=1$ this is $1$ if and only if $c_1=d_1=1$.



            Now let $x= c_1d_1...c_rd_r in H, y=c'_1d'_1...c'_sd'_s in K$ be nontrivial elements, with obvious notations, written in reduced form. The point will be that $[x,y]in Hcap K$ (this is obvious), and that, up to changing $x,y$ a bit, this can't be the trivial element.



            Now up to conjugation by some element of $C$, one may assume $d_r = 1$ and $c_1neq 1$ (this can be done by the hypothesis on $|C|$ - and $c_r neq 1$, but that follows from the reduced form); and up to conjugation by some element of $D$, $c'_1 = 1$ , $d_s'neq 1$ (using the cardinality hypothesis on $D$ - and $d_1' neq 1$, but again this follows from the reduced form).



            So with these hypotheses $[x,y] = c_1d_1...c_rd_1'....c_s'd_s'c_r^{-1}....d_1^{-1}c_1^{-1}d_s'^{-1}c_s'^{-1}...d_1'^{-1}$ which is written in reduced form, and is thus $neq 1$. Therefore $[x,y]in Hcap Ksetminus{1}$.



            Apply this to your supposed $Atimes {1}, {1}times B$ to get a contradiction.



            I don't know if there's an easier argument, or a not-too-complicated argument that encapsulates the low cardinality cases, but I guess for these you have to go "by hand" somehow; or perhaps you can adapt this argument to these cases by working a bit more. In any case I didn't want to bother with these cases, and this argument works in most cases and is pretty painless so in any case it's interesting to share







            share|cite|improve this answer














            share|cite|improve this answer



            share|cite|improve this answer








            edited 2 hours ago









            YCor

            7,528929




            7,528929










            answered 6 hours ago









            MaxMax

            14.3k11142




            14.3k11142























                3












                $begingroup$

                Let $f:Atimes Brightarrow C*D$ be an isomorphism where $A,B,C,D$ are non trivial groups. Let $a_0in A$ not trivial, for every $bin B$, $f(a_0)$ commutes with $f(b)$.



                Now we look the proof of the The Corollary 4.1.6 p. 187 of Magnus Karrass and Solitar.



                The first part of this proof asserts that if $f(a_0)$ is contained in the conjugated of a free factor, that is $f(a_0)$ is in $gCg^{-1}$ or is in $gDg^{-1}$ then so is $f(b)$ for every $bin B$. Without restricting the generality, we suppose that $f(a_0)$ and so $f(B)$ are contained in $gCg^{-1}$. Let $b_0$ a non trivial element of $B$ since $B$ is contained in the free conjugated factor $gCg^{-1}$ the same argument shows that $f(A)$ is contained in $gCg^{-1}$ this implies that $C*D$ is contained in a free conjugated factor. Contradiction.



                The second part of the proof shows that if $f(a_0)$ is not contained in a free conjugated factor, then there exists $u_c$ such $f(a_0)$ and $f(b)$ are a power of $u_c$.



                You can express $f(a_0)$ uniquely as a reduced sequence (Theorem 4.1.) this implies that there exists a unique element $u$ with minimal length such that $f(a_0)$ is a power of $u$ and if $f(a_0)$ is a power of $v$, $v$ is a power of $u$. This impllies that every element of $f(B)$ are power of $u$ and $f(B)$ and $B$ are cyclic. A similar argument shows that $f(A)$ and $A$ are cyclic, we deduce that $Atimes B$ is commutative. Contradicition since $C*D$ is not commutative.



                Reference.



                Combinatorial Group theory.



                Magnus, Karrass and Solitar.






                share|cite|improve this answer











                $endgroup$


















                  3












                  $begingroup$

                  Let $f:Atimes Brightarrow C*D$ be an isomorphism where $A,B,C,D$ are non trivial groups. Let $a_0in A$ not trivial, for every $bin B$, $f(a_0)$ commutes with $f(b)$.



                  Now we look the proof of the The Corollary 4.1.6 p. 187 of Magnus Karrass and Solitar.



                  The first part of this proof asserts that if $f(a_0)$ is contained in the conjugated of a free factor, that is $f(a_0)$ is in $gCg^{-1}$ or is in $gDg^{-1}$ then so is $f(b)$ for every $bin B$. Without restricting the generality, we suppose that $f(a_0)$ and so $f(B)$ are contained in $gCg^{-1}$. Let $b_0$ a non trivial element of $B$ since $B$ is contained in the free conjugated factor $gCg^{-1}$ the same argument shows that $f(A)$ is contained in $gCg^{-1}$ this implies that $C*D$ is contained in a free conjugated factor. Contradiction.



                  The second part of the proof shows that if $f(a_0)$ is not contained in a free conjugated factor, then there exists $u_c$ such $f(a_0)$ and $f(b)$ are a power of $u_c$.



                  You can express $f(a_0)$ uniquely as a reduced sequence (Theorem 4.1.) this implies that there exists a unique element $u$ with minimal length such that $f(a_0)$ is a power of $u$ and if $f(a_0)$ is a power of $v$, $v$ is a power of $u$. This impllies that every element of $f(B)$ are power of $u$ and $f(B)$ and $B$ are cyclic. A similar argument shows that $f(A)$ and $A$ are cyclic, we deduce that $Atimes B$ is commutative. Contradicition since $C*D$ is not commutative.



                  Reference.



                  Combinatorial Group theory.



                  Magnus, Karrass and Solitar.






                  share|cite|improve this answer











                  $endgroup$
















                    3












                    3








                    3





                    $begingroup$

                    Let $f:Atimes Brightarrow C*D$ be an isomorphism where $A,B,C,D$ are non trivial groups. Let $a_0in A$ not trivial, for every $bin B$, $f(a_0)$ commutes with $f(b)$.



                    Now we look the proof of the The Corollary 4.1.6 p. 187 of Magnus Karrass and Solitar.



                    The first part of this proof asserts that if $f(a_0)$ is contained in the conjugated of a free factor, that is $f(a_0)$ is in $gCg^{-1}$ or is in $gDg^{-1}$ then so is $f(b)$ for every $bin B$. Without restricting the generality, we suppose that $f(a_0)$ and so $f(B)$ are contained in $gCg^{-1}$. Let $b_0$ a non trivial element of $B$ since $B$ is contained in the free conjugated factor $gCg^{-1}$ the same argument shows that $f(A)$ is contained in $gCg^{-1}$ this implies that $C*D$ is contained in a free conjugated factor. Contradiction.



                    The second part of the proof shows that if $f(a_0)$ is not contained in a free conjugated factor, then there exists $u_c$ such $f(a_0)$ and $f(b)$ are a power of $u_c$.



                    You can express $f(a_0)$ uniquely as a reduced sequence (Theorem 4.1.) this implies that there exists a unique element $u$ with minimal length such that $f(a_0)$ is a power of $u$ and if $f(a_0)$ is a power of $v$, $v$ is a power of $u$. This impllies that every element of $f(B)$ are power of $u$ and $f(B)$ and $B$ are cyclic. A similar argument shows that $f(A)$ and $A$ are cyclic, we deduce that $Atimes B$ is commutative. Contradicition since $C*D$ is not commutative.



                    Reference.



                    Combinatorial Group theory.



                    Magnus, Karrass and Solitar.






                    share|cite|improve this answer











                    $endgroup$



                    Let $f:Atimes Brightarrow C*D$ be an isomorphism where $A,B,C,D$ are non trivial groups. Let $a_0in A$ not trivial, for every $bin B$, $f(a_0)$ commutes with $f(b)$.



                    Now we look the proof of the The Corollary 4.1.6 p. 187 of Magnus Karrass and Solitar.



                    The first part of this proof asserts that if $f(a_0)$ is contained in the conjugated of a free factor, that is $f(a_0)$ is in $gCg^{-1}$ or is in $gDg^{-1}$ then so is $f(b)$ for every $bin B$. Without restricting the generality, we suppose that $f(a_0)$ and so $f(B)$ are contained in $gCg^{-1}$. Let $b_0$ a non trivial element of $B$ since $B$ is contained in the free conjugated factor $gCg^{-1}$ the same argument shows that $f(A)$ is contained in $gCg^{-1}$ this implies that $C*D$ is contained in a free conjugated factor. Contradiction.



                    The second part of the proof shows that if $f(a_0)$ is not contained in a free conjugated factor, then there exists $u_c$ such $f(a_0)$ and $f(b)$ are a power of $u_c$.



                    You can express $f(a_0)$ uniquely as a reduced sequence (Theorem 4.1.) this implies that there exists a unique element $u$ with minimal length such that $f(a_0)$ is a power of $u$ and if $f(a_0)$ is a power of $v$, $v$ is a power of $u$. This impllies that every element of $f(B)$ are power of $u$ and $f(B)$ and $B$ are cyclic. A similar argument shows that $f(A)$ and $A$ are cyclic, we deduce that $Atimes B$ is commutative. Contradicition since $C*D$ is not commutative.



                    Reference.



                    Combinatorial Group theory.



                    Magnus, Karrass and Solitar.







                    share|cite|improve this answer














                    share|cite|improve this answer



                    share|cite|improve this answer








                    edited 5 hours ago

























                    answered 5 hours ago









                    Tsemo AristideTsemo Aristide

                    58k11445




                    58k11445























                        0












                        $begingroup$

                        To be a free product $Cast D$ implies the existence of an action on oriented tree such that $C$ is the stabilizer of some vertex $v_0$ and such that edge stabilizers are trivial (so that nontrivial element fix at most one vertex).



                        Let $(a,b)$ be a nontrivial element of $C$; we can suppose that $aneq 1$ up to switch $A$ and $B$. Then in the Bass-Serre tree, $(a,b)$ fixes a unique vertex $v_0$, and hence this vertex is also fixed by the centralizer of $(a,b)$, and hence $(a,1)$ fixes $v_0$. Applying this to $(a,1)$ shows that $B$ fixes the vertex $v_0$. Choose $1neq b'in B$. It fixes the unique vertex $v_0$ and hence its centralizer fixes $v_0$, so $A$ fixes $v_0$. Finally $G=Atimes B$ fixes $v_0$, so $D=1$.






                        share|cite|improve this answer









                        $endgroup$


















                          0












                          $begingroup$

                          To be a free product $Cast D$ implies the existence of an action on oriented tree such that $C$ is the stabilizer of some vertex $v_0$ and such that edge stabilizers are trivial (so that nontrivial element fix at most one vertex).



                          Let $(a,b)$ be a nontrivial element of $C$; we can suppose that $aneq 1$ up to switch $A$ and $B$. Then in the Bass-Serre tree, $(a,b)$ fixes a unique vertex $v_0$, and hence this vertex is also fixed by the centralizer of $(a,b)$, and hence $(a,1)$ fixes $v_0$. Applying this to $(a,1)$ shows that $B$ fixes the vertex $v_0$. Choose $1neq b'in B$. It fixes the unique vertex $v_0$ and hence its centralizer fixes $v_0$, so $A$ fixes $v_0$. Finally $G=Atimes B$ fixes $v_0$, so $D=1$.






                          share|cite|improve this answer









                          $endgroup$
















                            0












                            0








                            0





                            $begingroup$

                            To be a free product $Cast D$ implies the existence of an action on oriented tree such that $C$ is the stabilizer of some vertex $v_0$ and such that edge stabilizers are trivial (so that nontrivial element fix at most one vertex).



                            Let $(a,b)$ be a nontrivial element of $C$; we can suppose that $aneq 1$ up to switch $A$ and $B$. Then in the Bass-Serre tree, $(a,b)$ fixes a unique vertex $v_0$, and hence this vertex is also fixed by the centralizer of $(a,b)$, and hence $(a,1)$ fixes $v_0$. Applying this to $(a,1)$ shows that $B$ fixes the vertex $v_0$. Choose $1neq b'in B$. It fixes the unique vertex $v_0$ and hence its centralizer fixes $v_0$, so $A$ fixes $v_0$. Finally $G=Atimes B$ fixes $v_0$, so $D=1$.






                            share|cite|improve this answer









                            $endgroup$



                            To be a free product $Cast D$ implies the existence of an action on oriented tree such that $C$ is the stabilizer of some vertex $v_0$ and such that edge stabilizers are trivial (so that nontrivial element fix at most one vertex).



                            Let $(a,b)$ be a nontrivial element of $C$; we can suppose that $aneq 1$ up to switch $A$ and $B$. Then in the Bass-Serre tree, $(a,b)$ fixes a unique vertex $v_0$, and hence this vertex is also fixed by the centralizer of $(a,b)$, and hence $(a,1)$ fixes $v_0$. Applying this to $(a,1)$ shows that $B$ fixes the vertex $v_0$. Choose $1neq b'in B$. It fixes the unique vertex $v_0$ and hence its centralizer fixes $v_0$, so $A$ fixes $v_0$. Finally $G=Atimes B$ fixes $v_0$, so $D=1$.







                            share|cite|improve this answer












                            share|cite|improve this answer



                            share|cite|improve this answer










                            answered 2 hours ago









                            YCorYCor

                            7,528929




                            7,528929






























                                draft saved

                                draft discarded




















































                                Thanks for contributing an answer to Mathematics Stack Exchange!


                                • Please be sure to answer the question. Provide details and share your research!

                                But avoid



                                • Asking for help, clarification, or responding to other answers.

                                • Making statements based on opinion; back them up with references or personal experience.


                                Use MathJax to format equations. MathJax reference.


                                To learn more, see our tips on writing great answers.




                                draft saved


                                draft discarded














                                StackExchange.ready(
                                function () {
                                StackExchange.openid.initPostLogin('.new-post-login', 'https%3a%2f%2fmath.stackexchange.com%2fquestions%2f3110589%2fdoes-there-exist-a-group-that-is-both-a-free-product-and-a-direct-product-of-non%23new-answer', 'question_page');
                                }
                                );

                                Post as a guest















                                Required, but never shown





















































                                Required, but never shown














                                Required, but never shown












                                Required, but never shown







                                Required, but never shown

































                                Required, but never shown














                                Required, but never shown












                                Required, but never shown







                                Required, but never shown







                                Popular posts from this blog

                                How to reconfigure Docker Trusted Registry 2.x.x to use CEPH FS mount instead of NFS and other traditional...

                                is 'sed' thread safe

                                How to make a Squid Proxy server?